Circuito de resistencia que no es paralelo o serie

¿Cuál es la resistencia equivalente en este circuito (entre los puntos A y B)?ingrese la descripción de la imagen aquí

Puede resolver esto de la misma manera que puede resolver cualquier problema de resistencia: escriba las leyes de Kirchoff. Los trucos para la resistencia equivalente para circuitos en serie y en paralelo son atajos útiles, pero no proporcionan un algoritmo general para resolver estos problemas. Las leyes de Kirchoff sí.
Por lo general, cierro este tipo de preguntas, pero (a) llevas el tiempo suficiente como para que no tenga que estar contento con el botón de cerrar ;-) y (b) básicamente estás preguntando sobre un puente de Wheatstone. , que es más o menos el ejemplo canónico de un elemento de circuito que no puede reducirse mediante las reglas de serie y paralelo. Creo que preguntar cómo encontrar la resistencia de esa configuración es lo suficientemente aplicable en general como para que esté bien. Aunque todavía siento que la pregunta sería mejor con un poco más de explicación. (Tal vez solo soy yo)
David, deberías agregar el texto que creas necesario para mejorar la pregunta. Para mí, es solo una linda aplicación de las matemáticas. El propósito es señalar el comentario de kleingordon.

Respuestas (6)

Daré la respuesta a esta pregunta utilizando un método inusual que apareció en la sección de problemas del American Mathematical Monthly quizás a fines de la década de 1970. Esta no es necesariamente la manera fácil de resolver el problema, pero funciona muy bien desde un punto de vista algebraico.

La forma en que la mayoría de las personas resuelven la mayoría de los problemas de resistencia es usar reglas de resistencia en serie y en paralelo. Estos son matemáticamente elegantes en el sentido de que solo involucran resistencia. Pero este circuito no puede reducirse a reglas en serie y en paralelo (¿es esto cierto si escribe una serie infinita en R3, tal vez?), por lo que probablemente el método más directo sea aplicar un voltaje de V al circuito y usar álgebra para calcular la corriente total. Esto es poco elegante (pero físico) en el sentido de que introduce ideas distintas a la resistencia misma.

El método "delta" mencionado por Manishearth, (pero en este momento en realidad no funcionó hasta la respuesta final) es cómo un EE resolvería el problema. Tiene la ventaja de apegarse a la resistencia, pero implica cambios poco intuitivos en la topología del circuito.

El método que estoy dando aquí usa solo resistencias e ilustra una solución general para este tipo de problema. Si uno generaliza la R k a números complejos Z k , se puede usar para impedancias generales (al igual que el método delta), pero es más general que el método delta. También puede ayudar con la comprensión de los estudiantes de la resistencia de la hoja, por lo que creo que vale la pena escribirlo:


Primero, reemplazamos las resistencias con un material delgado y plano que tiene una " resistencia de hoja " de 1 ohmio por cuadrado. Con tal material, si recortamos un rectángulo de dimensiones 1 x R, obtendremos una resistencia de R ohmios entre dos conductores unidos a los lados de longitud 1:
Resistencia R ohm hecha de rectángulo Rx1

Ahora, lo que pasa con la resistencia de la hoja es que puede escalar la resistencia al tamaño que desee; siempre que mantenga la relación de las longitudes de los lados como "R", la resistencia resultante tendrá una resistencia R. La lámina puede estar formada por pequeñas láminas que se pegan entre sí. Para realizar el pegado correctamente, necesitamos utilizar cola aislante para las conexiones horizontales y cola conductora para las conexiones verticales. Esto se debe a que la corriente solo fluye de izquierda a derecha. Entonces, el pegamento aislante no ayuda ni dificulta el flujo de corriente, y las conexiones verticales no importan porque todo el pegamento conductor tiene el mismo voltaje de todos modos. Vi este método de cálculo de resistencias en una solución al problema E2459 en el American Mathematical Monthly, febrero de 1975 .

Entonces reemplace el circuito dado con uno donde cada resistencia se reemplaza por una región rectangular con dimensiones apropiadas para su resistencia. Al hacer esto, tenemos que hacer una suposición acerca de en qué dirección fluye la corriente a través de la resistencia R3. Asumiré que fluye de arriba hacia abajo. Y para establecer una escala para todo, hagamos que la dimensión vertical de R3 sea de longitud 1. Esto nos da el siguiente dibujo:
Circuito reescrito como resistencia de hoja rectangular

Ahora todo el circuito tiene una resistencia dada por la relación entre su largo y su ancho:

R = L / W = ( R 1 X 1 + R 2 X 2 ) / ( X 1 + X 4 )
Hay cuatro incógnitas, { X 1 , X 2 , X 4 , X 5 } . La comparación de dimensiones horizontales da dos ecuaciones independientes:
R 1 X 1 = R 4 X 4 + R 3 ,
R 5 X 5 = R 3 + R 2 X 2 .
Y comparando dimensiones verticales da:
X 1 = 1 + X 2 ,
X 5 = 1 + X 4 .

Esto elimina X 1 y X 5 para dar dos ecuaciones independientes con dos incógnitas:
R 1 + R 1 X 2 = R 4 X 4 + R 3 ,
R 5 + R 5 X 4 = R 3 + R 2 X 2 .
O:
R 1 X 2 R 4 X 4 = R 3 R 1 ,
R 2 X 2 + R 5 X 4 = R 3 R 5 .
Estos se resuelven para dar:
X 2 = R 3 R 5 R 1 R 5 + R 3 R 4 R 4 R 5 R 1 R 5 R 2 R 4 ,

X 4 = R 1 R 3 R 1 R 5 + R 2 R 3 R 1 R 2 R 1 R 5 R 2 R 4 ,

y entonces
X 1 = R 3 R 5 R 2 R 4 + R 3 R 4 R 4 R 5 R 1 R 5 R 2 R 4 ,

X 5 = R 1 R 3 R 2 R 4 + R 2 R 3 R 1 R 2 R 1 R 5 R 2 R 4 ,

Nosotros necesitamos W = X 1 + X 4 :
W = R 3 ( R 1 + R 2 + R 4 + R 5 ) ( R 1 + R 4 ) ( R 2 + R 5 ) R 1 R 5 R 2 R 4
y L = R 1 X 1 + R 2 X 2 :
L = R 3 ( R 4 + R 5 ) ( R 1 + R 2 ) R 1 R 2 ( R 4 + R 5 ) R 4 R 5 ( R 1 + R 2 ) R 1 R 5 R 2 R 4
entonces la resistencia total es:
R = L / W = R 1 R 4 ( R 2 + R 5 ) + R 2 R 5 ( R 1 + R 4 ) R 3 ( R 4 + R 5 ) ( R 1 + R 2 ) ( R 2 + R 5 ) ( R 1 + R 4 ) R 3 ( ( R 1 + R 2 ) + ( R 4 + R 5 ) ) .
En lo anterior, he agrupado términos para dejar claro que esto da la respuesta correcta en el límite de R 3 va a 0 o .

Pero yo pienso R 1 X 1 = R 4 X 4 R 3 y R 5 X 5 = R 2 X 2 R 3 . ¿Qué estoy haciendo mal? Por favor explique.

Use una transformada estrella-triángulo para simplificar parte del circuito. También puede utilizar el principio de superposición.

Originalmente había visto esto en un problema matemático que decía "¿cuál es la cantidad mínima de resistencias necesarias para obtener una resistencia de pi con una precisión de 1 parte en un millón?" La solución allí transformó el problema en uno de teselar una matriz rectangular con un número mínimo de teselas cuadradas (de tamaño arbitrario) con la relación entre la longitud y el ancho siendo una aproximación entera de pi. Traducido nuevamente a un circuito, el circuito ganador fue de esta forma (con las resistencias cada una un pequeño múltiplo de un ohmio).
@CarlBrannen Hmm... Ese problema que trataría de resolver con una serie infinita de resistencias. ζ ( 2 ) aparece en mi mente, ya que es relativamente simple de construir con resistencias integrales, pero desafortunadamente finalmente obtendrá una resistencia de 6 / π 2 . Teselar seis de estos en paralelo te da 1 / π 2 . Dudo que las resistencias puedan hacer cosas de raíz cuadrada.
O simplemente tome un material de resistividad constante + sección transversal, dibuje un (semi) círculo con radio = longitud de 1 Ω resistor. Coloque su material en este círculo. =D
Otra forma de hacerlo sería utilizar la expansión de arcán X , pero tiene aspectos negativos.
@CarlBrannen, ¿por qué no hacer eso como una pregunta de 'rompecabezas' por separado?
@nibot Eso sería bueno, aunque no sé cuál es la política de P.SE sobre preguntas de rompecabezas (que generarán muchas respuestas diferentes).
@CarlBrannen: Esto es desconcertante, ya que puede realizar una expansión de fracción continua usando resistencias en serie y en paralelo, y esta es una forma extremadamente económica de producir pi. Solo necesitaría tantas resistencias de 1 ohm como la suma de los denominadores de fracción continua hasta la precisión requerida. Pi tiene un denominador grande desde el principio, por lo que este podría ser el punto de conflicto.
@RonMaimon ahora, ¿por qué no pensé en eso? Sí, la fracción continua uno parece fácil de hacer, pero el diagrama puede ser un poco complicado...
 A x----x-----[1]-----x-----[2]-----x----x B
        |             |             |
       [4]           [3]           [5]
        |             |             |
        |-------------x-------------|    

Figura 1. Circuito original de OP.

Como sugiere Manishearth, se puede realizar una Y - Δ transformar de Y -resistencias R 1 , R 2 y R 3 , a Δ -conductancias GRAMO 1 , GRAMO 2 y GRAMO 3 (usando un 123 convención de etiquetado simétrico), cf. Fig. 2 a continuación.

 A x----x------x-----[3]-----x------x----x B
        |      |             |      |
       [4]    [2]           [1]    [5]
        |      |             |      |
        |------x-------------x------|    

Figura 2. A Δ -circuito equivalente al circuito original de OP.

En cuanto a las fórmulas, el Y - Δ la transformación se da como

GRAMO i   :=   R i j = 1 3 R j k = 1 3 R k   =   R i R 1 + R 2 + R 3 R 1 R 2 R 3 , i = 1 , 2 , 3.

los Δ -El circuito equivalente de la figura 2 puede verse como compuesto únicamente de resistencias en serie y en paralelo . La conductancia equivalente entre A y B por lo tanto se convierte

1 R   =   GRAMO 3 + 1 1 GRAMO 2 + 1 R 4 + 1 GRAMO 1 + 1 R 5 .

(Finalmente mencionemos que también es posible aplicar el Y - Δ transformar a otros triples de las cinco resistencias que 123 .)

Así es como lo haría, siguiendo el método descrito por kleingordon en un comentario. Este método es menos genial pero más general que la respuesta de Carl Brannen, porque funcionará incluso en el caso de que haya cables cruzados y no pueda reorganizarlos en una sola hoja de material resistivo.

Sea el potencial eléctrico en A ser V A y eso en B ser V B . Además, deje que el potencial en el cable que conecta R 1 a R 2 y R 3 ser V C y deje que el potencial en el cable que conecta R 4 a R 3 y R 5 ser V D . Sabemos que la corriente a través de cada resistencia debe ser igual a la diferencia de potencial dividida por la resistencia, por lo que tenemos

yo 1 = R 1 ( V A V C )
yo 2 = R 2 ( V C V B )
yo 3 = R 3 ( V C V D )
yo 4 = R 4 ( V A V D )
yo 5 = R 5 ( V D V B ) .

También sabemos que la corriente debe conservarse en cada cruce, lo que nos da

yo 1 + yo 2 = yo 4 + yo 5
yo 1 = yo 2 + yo 3
yo 4 + yo 3 = yo 5 ,
pero la última de estas tres ecuaciones es redundante porque puede derivarse de las otras dos, por lo que hay siete ecuaciones en total, en nueve incógnitas (cinco corrientes y cuatro potenciales).

Queremos calcular la resistencia, que viene dada por ( V A V B ) / ( yo 1 + yo 2 ) . Dado que todo es lineal, podemos suponer sin pérdida de generalidad que V B = 0 y V A = 1 . Esto nos da siete ecuaciones en siete incógnitas, que podemos resolver para encontrar la respuesta.

No lo he trabajado porque es un poco laborioso (probablemente usaría un sistema de álgebra computacional en lugar de hacerlo a mano), pero debería dar la misma respuesta que el método de Carl Brannen.

Siguiendo con el comentario de un Googler a la respuesta de Carl Brannen:

Pero yo pienso R 1 X 1 = R 4 X 4 R 3 y R 5 X 5 = R 2 X 2 R 3 . ¿Qué estoy haciendo mal? Por favor explique

Si sigue esta corrección (es decir, cambia sus subíndices 1 y 4, y 2 y 5 en su consideración horizontal de apertura; no es necesario cambiar las declaraciones verticales), entonces obtendrá un resultado similar a:

R = L / W = R 1 R 4 ( R 2 + R 5 ) + R 2 R 5 ( R 1 + R 4 ) R 3 ( R 4 + R 5 ) ( R 1 + R 2 ) ( R 2 + R 5 ) ( R 1 + R 4 ) R 3 ( ( R 1 + R 2 ) + ( R 4 + R 5 ) ) .

pero sin los signos negativos de cada uno R 3 término:

R = L / W = R 1 R 4 ( R 2 + R 5 ) + R 2 R 5 ( R 1 + R 4 ) + R 3 ( R 4 + R 5 ) ( R 1 + R 2 ) ( R 2 + R 5 ) ( R 1 + R 4 ) + R 3 ( ( R 1 + R 2 ) + ( R 4 + R 5 ) ) .

Este resultado también da los resultados correctos tanto para R que tiende a 0 como para R que tiende a infinito, pero las definiciones de las R ahora son consistentes con el diagrama.

Aquí hay algunos pasos:

Tenemos:

R 1 X 1 = R 4 X 4 R 3 ,

R 5 X 5 = R 2 X 2 R 3 .

Usando también:

X 1 = X 2 + 1 ,

X 5 = X 4 + 1 ,

y eliminando X 1 y X 5 de las ecuaciones horizontales, obtenemos:

R 1 X 2 + R 1 = R 4 X 4 R 3 ,

R 2 X 2 R 3 = R 5 X 4 + R 5 .

Estos se resuelven para dar:

X 4 = R 1 R 5 + R 1 R 3 + R 1 R 2 + R 2 R 3 R 2 R 4 R 1 R 5

X 2 = R 1 R 5 + R 3 R 5 + R 3 R 4 + R 4 R 5 R 2 R 4 R 1 R 5

X 1 = R 2 R 4 + R 3 R 5 + R 3 R 4 + R 4 R 5 R 2 R 4 R 1 R 5

X 5 = R 1 R 3 + R 1 R 2 + R 2 R 3 + R 2 R 4 R 2 R 4 R 1 R 5

W = R 3 ( R 1 + R 2 + R 4 + R 5 ) + ( R 2 + R 5 ) ( R 1 + R 4 ) R 2 R 4 R 1 R 5

L = R 3 ( R 1 + R 2 ) ( R 4 + R 5 ) + R 1 R 4 ( R 2 + R 5 ) + R 2 R 5 ( R 1 + R 4 ) R 2 R 4 R 1 R 5

y finalmente

R = L / W = R 1 R 4 ( R 2 + R 5 ) + R 2 R 5 ( R 1 + R 4 ) + R 3 ( R 4 + R 5 ) ( R 1 + R 2 ) ( R 2 + R 5 ) ( R 1 + R 4 ) + R 3 ( ( R 1 + R 2 ) + ( R 4 + R 5 ) ) .

Curiosamente, si las resistencias R 1 , R 2 , R 4 , R 5 todos tienen el mismo valor, digamos ρ , entonces se puede demostrar que la resistencia de todo el circuito no depende de R 3 en absoluto y, en cambio, será igual a ρ .

"Curiosamente, si todos excepto R 3 tener el mismo valor ρ entonces se puede demostrar que la resistencia no depende de R 3 en absoluto ": esto se puede ver intuitivamente ya que el circuito es simétrico y, por lo tanto, no fluye corriente a través de R 3 por lo que la resistencia es básicamente innecesaria en el circuito. Luego se simplifica a un simple par de resistencias en paralelo. Esto se explota en el puente de Wheatstone.

Creo que esta fórmula podría ayudarte:

R 3 R 1 + 3 R 2 R 4 + 3 R 5

Intente agregar detalles adicionales a su respuesta
@ArnavMahajan ¿Qué tipo de detalles adicionales necesita?